Những câu hỏi liên quan
Văn Thắng Hồ
Xem chi tiết
Tiến Nguyễn Minh
Xem chi tiết
Vũ Tiến Manh
21 tháng 10 2019 lúc 22:19

1) Áp dụng bunhiacopxki ta được \(\sqrt{\left(2a^2+b^2\right)\left(2a^2+c^2\right)}\ge\sqrt{\left(2a^2+bc\right)^2}=2a^2+bc\), tương tự với các mẫu ta được vế trái \(\le\frac{a^2}{2a^2+bc}+\frac{b^2}{2b^2+ac}+\frac{c^2}{2c^2+ab}\le1< =>\)\(1-\frac{bc}{2a^2+bc}+1-\frac{ac}{2b^2+ac}+1-\frac{ab}{2c^2+ab}\le2< =>\)

\(\frac{bc}{2a^2+bc}+\frac{ac}{2b^2+ac}+\frac{ab}{2c^2+ab}\ge1\)<=> \(\frac{b^2c^2}{2a^2bc+b^2c^2}+\frac{a^2c^2}{2b^2ac+a^2c^2}+\frac{a^2b^2}{2c^2ab+a^2b^2}\ge1\)  (1) 

áp dụng (x2 +y2 +z2)(m2+n2+p2\(\ge\left(xm+yn+zp\right)^2\)

(2a2bc +b2c2 + 2b2ac+a2c2 + 2c2ab+a2b2). VT\(\ge\left(bc+ca+ab\right)^2\)   <=> (ab+bc+ca)2. VT \(\ge\left(ab+bc+ca\right)^2< =>VT\ge1\)  ( vậy (1) đúng)

dấu '=' khi a=b=c

Bình luận (0)
 Khách vãng lai đã xóa
HD Film
21 tháng 10 2019 lúc 22:26

4b, \(\frac{a^3}{a^2+b^2}+\frac{b^3}{b^2+c^2}+\frac{c^3}{c^2+a^2}=1-\frac{ab^2}{a^2+b^2}+1-\frac{bc^2}{b^2+c^2}+1-\frac{ca^2}{a^2+c^2}\)

\(\ge3-\frac{ab^2}{2ab}-\frac{bc^2}{2bc}-\frac{ca^2}{2ac}=3-\frac{\left(a+b+c\right)}{2}=\frac{3}{2}\)

Bình luận (0)
 Khách vãng lai đã xóa
HD Film
21 tháng 10 2019 lúc 22:35

4c, 

\(\frac{a+1}{b^2+1}+\frac{b+1}{c^2+1}+\frac{c+1}{a^2+1}=a+b+c-\frac{b^2}{b^2+1}-\frac{c^2}{c^2+1}-\frac{a^2}{a^2+1}+3--\frac{b^2}{b^2+1}-\frac{c^2}{c^2+1}-\frac{a^2}{a^2+1}\)\(\ge6-2\cdot\frac{\left(a+b+c\right)}{2}=3\)

Bình luận (0)
 Khách vãng lai đã xóa
Nguyễn Quốc Gia Huy
Xem chi tiết
alibaba nguyễn
22 tháng 8 2017 lúc 7:06

Ta có:

\(\frac{1}{\left(2a+b+c\right)^2}+\frac{1}{\left(a+2b+c\right)^2}+\frac{1}{\left(a+b+2c\right)^2}\)

\(\le\frac{1}{4\left(a+b\right)\left(a+c\right)}+\frac{1}{4\left(b+a\right)\left(b+c\right)}+\frac{1}{4\left(c+a\right)\left(c+b\right)}\)

\(=\frac{2\left(a+b+c\right)}{4\left(a+b\right)\left(b+c\right)\left(c+a\right)}\)

\(=\frac{a+b+c}{2\left(a+b\right)\left(b+c\right)\left(c+a\right)}\)

Giờ ta cần chứng minh

\(\frac{a+b+c}{2\left(a+b\right)\left(b+c\right)\left(c+a\right)}\le\frac{9}{16\left(ab+bc+ca\right)}\)

\(\Leftrightarrow\left(a+b\right)\left(b+c\right)\left(c+a\right)\ge\frac{8}{9}\left(a+b+c\right)\left(ab+bc+ca\right)\)

Ta có:

\(\left(a+b\right)\left(b+c\right)\left(c+a\right)=\left(a+b+c\right)\left(ab+bc+ca\right)-3abc\)

\(\ge\left(a+b+c\right)\left(ab+bc+ca\right)-\frac{1}{9}\left(a+b+c\right)\left(ab+bc+ca\right)\)

\(=\frac{8}{9}\left(a+b+c\right)\left(ab+bc+ca\right)\)

Vậy ta có ĐPCM

Bình luận (0)
Nguyễn Thiều Công Thành
Xem chi tiết
Agami Raito
Xem chi tiết
Nguyễn Việt Lâm
24 tháng 5 2019 lúc 13:46

\(a+b+c=6abc\Leftrightarrow\frac{1}{ab}+\frac{1}{ac}+\frac{1}{bc}=6\)

Đặt \(\left\{{}\begin{matrix}\frac{1}{a}=x\\\frac{1}{b}=y\\\frac{1}{c}=z\end{matrix}\right.\) \(\Rightarrow xy+xz+yz=6\)

\(P=\sum\frac{\frac{1}{yz}}{\frac{1}{x^3}\left(\frac{1}{z}+\frac{2}{y}\right)}=\sum\frac{x^3}{y+2z}=\sum\frac{x^4}{xy+2xz}\ge\frac{\left(x^2+y^2+z^2\right)^2}{3\left(xy+xz+yz\right)}\ge\frac{\left(xy+xz+yz\right)^2}{3\left(xy+xz+yz\right)}=2\)

Dấu "=" xảy ra khi \(x=y=z=\sqrt{2}\Leftrightarrow a=b=c=\frac{1}{\sqrt{2}}\)

Bình luận (0)
Giao Khánh Linh
Xem chi tiết
Lê Ánh
Xem chi tiết
Phước Nguyễn
26 tháng 3 2016 lúc 17:20

Chịu bài này rồi!

Bình luận (0)
Trần Thùy Trang
26 tháng 3 2016 lúc 17:34

mk mới hk lp 6 , bài này bó tay ko giải đc

Bình luận (0)
Lê Ánh
26 tháng 3 2016 lúc 18:49

Đề thi HSG á :v

Bình luận (0)
Neet
Xem chi tiết
Akai Haruma
2 tháng 3 2017 lúc 0:34

Bài 3)

BĐT cần chứng minh tương đương với:

\(\left ( \frac{a}{a+b} \right )^2+\left ( \frac{b}{b+c} \right )^2+\left ( \frac{c}{c+a} \right )^2\geq \frac{1}{2}\left ( 3-\frac{a}{a+b}-\frac{b}{b+c}-\frac{c}{c+a} \right )\)

Để cho gọn, đặt \((x,y,z)=\left (\frac{b}{a},\frac{c}{b},\frac{a}{c}\right)\) \(\Rightarrow xyz=1\).

BĐT được viết lại như sau:

\(A=2\left [ \frac{1}{(x+1)^2}+\frac{1}{(y+1)^2}+\frac{1}{(z+1)^2} \right ]+\frac{1}{x+1}+\frac{1}{y+1}+\frac{1}{z+1}\geq 3\) \((\star)\)

Ta nhớ đến hai bổ đề khá quen thuộc sau:

Bổ đề 1: Với \(a,b>0\) thì \(\frac{1}{(a+1)^2}+\frac{1}{(b+1)^2}\geq \frac{1}{ab+1}\)

Cách CM rất đơn giản, Cauchy - Schwarz:

\((a+1)^2\leq (a+b)(a+\frac{1}{b})\Rightarrow \frac{1}{(a+1)^2}\geq \frac{b}{(a+b)(ab+1)}\)

Tương tự với biểu thức còn lại và cộng vào thu được đpcm

Bổ đề 2: Với \(x,y>0,xy\geq 1\) thì \(\frac{1}{x^2+1}+\frac{1}{y^2+1}\geq \frac{2}{xy+1}\)

Cách CM: Quy đồng ta có đpcm.

Do tính hoán vị nên không mất tổng quát giả sử \(z=\min (x,y,z)\)

\(\Rightarrow xy\geq 1\). Áp dụng hai bổ đề trên:

\(A\geq 2\left [ \frac{1}{xy+1}+\frac{1}{(z+1)^2} \right ]+\frac{2}{\sqrt{xy}+1}+\frac{1}{z+1}=2\left [ \frac{z}{z+1}+\frac{1}{(z+1)^2} \right ]+\frac{2\sqrt{z}}{\sqrt{z}+1}+\frac{1}{z+1}\)

\(\Leftrightarrow A\geq \frac{2(z^2+z+1)}{(z+1)^2}+\frac{1}{z+1}+2-\frac{2}{\sqrt{z}+1}\geq 3\)

\(\Leftrightarrow 2\left [ \frac{z^2+z+1}{(z+1)^2}-\frac{3}{4} \right ]+\frac{1}{z+1}-\frac{1}{2}-\left ( \frac{2}{\sqrt{z}+1}-1 \right )\geq 0\)

\(\Leftrightarrow \frac{(z-1)^2}{2(z+1)^2}-\frac{z-1}{2(z+1)}+\frac{z-1}{(\sqrt{z}+1)^2}\geq 0\Leftrightarrow (z-1)\left [ \frac{1}{(\sqrt{z}+1)^2}-\frac{1}{(z+1)^2} \right ]\geq 0\)

\(\Leftrightarrow \frac{\sqrt{z}(\sqrt{z}-1)^2(\sqrt{z}+1)(z+\sqrt{z}+2)}{(\sqrt{z}+1)^2(z+1)^2}\geq 0\) ( luôn đúng với mọi \(z>0\) )

Do đó \((\star)\) được cm. Bài toán hoàn tất.

Dấu bằng xảy ra khi \(a=b=c\)

P/s: Nghỉ tuyển lâu rồi giờ mới gặp mấy bài BĐT phải động não. Khuya rồi nên xin phép làm bài 3 trước. Hai bài kia xin khiếu. Nếu làm đc chắc tối mai sẽ post.

Bình luận (1)
Lightning Farron
2 tháng 3 2017 lúc 18:11

Bài 1:

Cho \(a=b=c=\dfrac{1}{\sqrt{3}}\). Khi đó \(M=\sqrt{3}-2\)

Ta sẽ chứng minh nó là giá trị nhỏ nhất

Thật vậy, đặt c là giá trị nhỏ nhất của a,b,c. Khi đó, ta cần chứng minh

\(\frac{a^2}{b}+\frac{b^2}{c}+\frac{c^2}{a}-\frac{2(a^2+b^2+c^2)}{\sqrt{ab+ac+bc}}\geq(\sqrt3-2)\sqrt{ab+ac+bc}\)

\(\Leftrightarrow\sqrt{ab+ac+bc}\left(\frac{a^2}{b}+\frac{b^2}{c}+\frac{c^2}{a}-\sqrt{3(ab+ac+bc)}\right)\geq2(a^2+b^2+c^2-ab-ac-bc)\)

\(\Leftrightarrow\frac{a^2}{b}+\frac{b^2}{a}-a-b+\frac{b^2}{c}+\frac{c^2}{a}-\frac{b^2}{a}-c+a+b+c-\sqrt{3(ab+ac+bc)}\geq\)

\(\geq2((a-b)^2+(c-a)(c-b))\)

\(\Leftrightarrow(a-b)^2\left(\frac{1}{a}+\frac{1}{b}-2\right)+(c-a)(c-b)\left(\frac{1}{a}+\frac{b}{ac}-2\right)+a+b+c-\sqrt{3(ab+ac+bc)}\geq0\)

Đúng bởi \(\frac{1}{a}+\frac{1}{b}-2>0;\frac{1}{a}+\frac{b}{ac}-2\geq\frac{1}{a}+\frac{1}{a}-2>0\)

\(a+b+c-\sqrt{3(ab+ac+bc)}=\frac{(a-b)^2+(c-a)(c-b)}{a+b+c+\sqrt{3(ab+ac+bc)}}\geq0\)

BĐT đã được c/m. Vậy \(M_{Min}=\sqrt{3}-2\Leftrightarrow a=b=c=\dfrac{1}{\sqrt{3}}\)

P/s: Nhìn qua thấy ngon mà làm mới thấy thật sự là "choáng"

Bình luận (3)
Hung nguyen
2 tháng 3 2017 lúc 10:57

Câu 1/ Ta có

\(ab+bc+ca\le\frac{\left(a+b+c\right)^2}{3}\)

\(\Leftrightarrow1\le\frac{\left(a+b+c\right)^2}{3}\)

\(\Leftrightarrow\left(a+b+c\right)^2\ge3\)

\(\Leftrightarrow\sqrt{3}\le a+b+c< 3\)

Ta có: \(M=\frac{a^2\left(1-2b\right)}{b}+\frac{b^2\left(1-2c\right)}{c}+\frac{c^2\left(1-2a\right)}{a}\)

\(=\frac{a^2}{b}+\frac{b^2}{c}+\frac{c^2}{a}-2\left(a^2+b^2+c^2\right)\)

\(\ge\frac{\left(a+b+c\right)^2}{a+b+c}-2\left(a^2+b^2+c^2\right)-4\left(ab+bc+ca\right)+4\left(ab+bc+ca\right)\)

\(=a+b+c-2\left(a+b+c\right)^2+4\) (1)

Đặt \(a+b+c=x\left(\sqrt{3}\le x< 3\right)\)

Ta tìm GTNN của hàm số: \(y=-2x^2+x+4\)

\(\Rightarrow y'=-4x+1=0\)

\(\Rightarrow x=\frac{1}{4}=0,25\)

Thế x lần lược các giá trị \(\left\{\begin{matrix}x=0,25\\x=\sqrt{3}\end{matrix}\right.\)

\(\Rightarrow\left\{\begin{matrix}y=4,125\\y=-2+\sqrt{3}\end{matrix}\right.\)

\(\Rightarrow y_{min}=-2+\sqrt{3}\) đạt cực trị tại \(x=\sqrt{3}\) (2)

Từ (1) và (2) ta suy ra GTNN của M là \(-2+\sqrt{3}\) tại \(a=b=c=\frac{1}{\sqrt{3}}\)

Bình luận (7)
Fire Sky
Xem chi tiết
tth_new
14 tháng 11 2019 lúc 13:46

\(\Leftrightarrow\Sigma_{cyc}\frac{\left(ab+bc+ca\right)^2}{2a^2+bc}\le\left(a+b+c\right)^2\)

Ta có: \(\frac{\left(ab+bc+ca\right)^2}{2a^2+bc}\le\frac{\left(ab+ca\right)^2}{2a^2}+\frac{\left(bc\right)^2}{bc}=\frac{\left(b+c\right)^2}{2}+bc\)

Tương tự rồi cộng lại ta thu được:

\(L.H.S\le\frac{\left(a+b\right)^2+\left(b+c\right)^2+\left(c+a\right)^2}{2}+ab+bc+ca\)

\(=\frac{2\left(a^2+b^2+c^2\right)+2\left(ab+bc+ca\right)}{2}+ab+bc+ca\)\(=\left(a+b+c\right)^2\)

P/s: Nhìn đơn giản chứ nó là bao nhiêu ngày suy nghĩ đấy ạ:( Chả biết đúng hay sai nữa:v

Bình luận (0)
 Khách vãng lai đã xóa